Difference between revisions of "2017 AMC 10A Problems/Problem 6"

(Problem)
Line 3: Line 3:
  
 
<math>\text{(A) 16}\qquad\text{(B) 17}\qquad\text{(C) 18}\qquad\text{(D) 19}\qquad\text{(E) 20}</math>
 
<math>\text{(A) 16}\qquad\text{(B) 17}\qquad\text{(C) 18}\qquad\text{(D) 19}\qquad\text{(E) 20}</math>
 +
 +
Someone please fix this. This is problem number 10, but number 6 on amc 12
  
 
==Solution==
 
==Solution==

Revision as of 15:55, 8 February 2017

Problem

Joy has $30$ thin rods, one each of every integer length from $1$ cm through $30$ cm. She places the rods with lengths $3$ cm, $7$ cm, and $15$ cm on a table. She then wants to choose a fourth rod that she can put with these three to form a quadrilateral with positive area. How many of the remaining rods can she choose as the fourth rod?

$\text{(A) 16}\qquad\text{(B) 17}\qquad\text{(C) 18}\qquad\text{(D) 19}\qquad\text{(E) 20}$

Someone please fix this. This is problem number 10, but number 6 on amc 12

Solution